Magnetic field generated by a current in a wire - special relativity

In summary, the conversation discusses the use of Gauss' theorem to calculate the electric field in the primed frame, which is then transformed to the unprimed frame using Lorentz transformations. There is a discussion about the signs in the transformed equations and the introduction of a static negative charge distribution to cancel out the electric field in the unprimed frame. It is noted that the surface does not undergo contraction and the addition of the negative charge distribution does not affect the magnetic field in the unprimed frame.
  • #1
Frostman
115
17
Homework Statement
Calculate the magnetic field ##\underline B## generated by a current ##I## in an infinitely long straight wire, knowing the shape of the electric field generated by a straight-line and uniform distribution of static electric charges, the transformation laws of electromagnetic fields for Lorentz boost, and appropriately using the superposition principle for electromagnetic fields.
Relevant Equations
Gauss theorem
Lorentz transformations for current density and electromagnetic fields
WhatsApp Image 2021-06-22 at 11.56.52.jpeg


First I wrote in ##S'##, by using Gauss theorem
$$
\int_{\Sigma} \underline E' \cdot \hat n d\Sigma = \frac Q {\varepsilon_0} \rightarrow E'(r)2\pi rH=\frac{\lambda'H}{\varepsilon_0}
$$
$$
\underline E'(\underline r)=\frac{\lambda'}{2\pi\varepsilon_0r}\hat r
$$
Its components are:

##E_x'=\frac{\lambda'}{2\pi\varepsilon_0r}\cos\theta##
##E_y'=\frac{\lambda'}{2\pi\varepsilon_0r}\sin\theta##
##E_z'=0##

Now I notice that ##Q=\rho'HA## and ##\lambda'=\frac{Q}{H}##, so ##\rho'=\frac{\lambda'}{A}##

With this information, I can made ##J'\mu=(\rho', 0, 0, 0)## and consider a ##S## frame moving respect ##S'## with a speed ##\underline v=- v\hat z##. For a Lorentz boost we have:

##\rho = \gamma(\rho'-vJ_z')=\gamma\rho'##
##J_x=J_x'=0##
##J_y=J_y'=0##
##J_z=\gamma(J_z'-v\rho')=-\gamma v\rho'##

Now we have ##J\mu=(\gamma\rho', 0, 0, -\gamma v \rho')##, it appears a current density in ##S## frame. I evaluate the new fields:

##E_x=\gamma(E_x'+vB_y')=\gamma E_x' = \gamma \frac{\lambda'}{2\pi\varepsilon_0r}\cos\theta##
##E_x=\gamma(E_y'-vB_x')=\gamma E_y' = \gamma \frac{\lambda'}{2\pi\varepsilon_0r}\sin\theta##
##E_z=E_z'=0##
##B_x=\gamma(B_x'-vE_y')=-\gamma v E_y' = \gamma v \frac{\lambda'}{2\pi\varepsilon_0r}\sin\theta = \frac{J_zA}{2\pi\varepsilon_0r}\sin\theta=\frac{I}{2\pi\varepsilon_0r}\sin\theta##
##B_y=\gamma(B_y'-vE_x')=\gamma v E_x' = \gamma v \frac{\lambda'}{2\pi\varepsilon_0r}\cos\theta = \frac{-J_zA}{2\pi\varepsilon_0r}\cos\theta=- \frac{I}{2\pi\varepsilon_0r}\cos\theta##
##B_z=B_z'=0##

I observe that it shows an electric field in ##S## frame that we should delete, and we do this by using superposition principle for electromagnetic fields introducing a linear charge density that delete ##E_x## and ##E_y##, and its value is ##\tilde{\lambda} = -\gamma \lambda'##.

I hope that everything is right also at the level of signs given the reference systems I used, I shouldn't have mistaken the Lorentz transformations.

I have a doubt, beyond what I have done. Surface ##A## does not undergo any contraction since the motion is orthogonal to the surface, right?

The moment I introduce a new linear charge density ##\tilde{\lambda}##, the magnetic field becomes more intense due to the fact that more current is added, especially it should double right?
 
  • Like
Likes Delta2
Physics news on Phys.org
  • #2
Frostman said:
With this information, I can made ##J'\mu=(\rho', 0, 0, 0)## and consider a ##S## frame moving respect ##S'## with a speed ##\underline v=- v\hat z##. For a Lorentz boost we have:

##\rho = \gamma(\rho' - vJ_z')=\gamma\rho'##
##J_x=J_x'=0##
##J_y=J_y'=0##
##J_z=\gamma(J_z'-v\rho')=-\gamma v\rho'##
I believe the minus signs in the first and last equations above should be plus signs. You are transforming from the primed frame to the unprimed frame. Note that if ##\rho'## is positive, you would expect ##j_z## to also be positive since the positively charged cylinder is moving in the positive z-direction relative to the unprimed frame.

Frostman said:
Now we have ##J\mu=(\gamma\rho', 0, 0, -\gamma v \rho')##, it appears a current density in ##S## frame.
##j_z## will not have the minus sign

Frostman said:
I evaluate the new fields:

##E_x=\gamma(E_x'+vB_y')=\gamma E_x' = \gamma \frac{\lambda'}{2\pi\varepsilon_0r}\cos\theta##
OK

Frostman said:
##E_x=\gamma(E_y'-vB_x')=\gamma E_y' = \gamma \frac{\lambda'}{2\pi\varepsilon_0r}\sin\theta##
##E_x## on the far left should be ##E_y##. Otherwise OK.

Frostman said:
##B_x=\gamma(B_x'-vE_y')=-\gamma v E_y' = \gamma v \frac{\lambda'}{2\pi\varepsilon_0r}\sin\theta = \frac{J_zA}{2\pi\varepsilon_0r}\sin\theta=\frac{I}{2\pi\varepsilon_0r}\sin\theta##
The minus sign after the second "##=##" should carry over to the expression after the third "##=##". Check to see if this will change the sign of the final expression in terms of ##I##.

Frostman said:
##B_y=\gamma(B_y'-vE_x')=\gamma v E_x' = \gamma v \frac{\lambda'}{2\pi\varepsilon_0r}\cos\theta = \frac{-J_zA}{2\pi\varepsilon_0r}\cos\theta=- \frac{I}{2\pi\varepsilon_0r}\cos\theta##
I'll let you double-check the signs for this equation.

Frostman said:
I observe that it shows an electric field in ##S## frame that we should delete, and we do this by using superposition principle for electromagnetic fields introducing a linear charge density that delete ##E_x## and ##E_y##, and its value is ##\tilde{\lambda} = -\gamma \lambda'##.
Yes, you can superpose this static negative charge distribution in the unprimed frame to get rid of the electric field in the unprimed frame.

Frostman said:
I have a doubt, beyond what I have done. Surface ##A## does not undergo any contraction since the motion is orthogonal to the surface, right?
Right

Frostman said:
The moment I introduce a new linear charge density ##\tilde{\lambda}##, the magnetic field becomes more intense due to the fact that more current is added, especially it should double right?
No. you are superposing a static negative charge distribution in the unprimed frame.
It will not affect the B-field that you have calculated in the unprimed frame.
 
  • Like
Likes Frostman

What is a magnetic field?

A magnetic field is a region in space where a magnetic force can be detected. It is created by moving electric charges, such as electrons, and can exert a force on other moving charges.

How is a magnetic field generated by a current in a wire?

When an electric current flows through a wire, it creates a circular magnetic field around the wire. This is known as the right-hand rule, where the direction of the magnetic field is determined by the direction of the current flow and the direction of the curl of the fingers of the right hand.

What is special relativity and how does it relate to magnetic fields?

Special relativity is a theory that explains the relationship between space and time. It states that the laws of physics are the same for all observers in uniform motion. In the context of magnetic fields, special relativity explains how the strength of the magnetic field changes for observers moving at different speeds.

How does special relativity affect the strength of a magnetic field?

Special relativity predicts that as an observer moves closer to the speed of light, the strength of the magnetic field will increase. This is due to the fact that as an object approaches the speed of light, its mass increases and thus its magnetic field becomes stronger.

Can special relativity explain all aspects of magnetic fields?

No, special relativity is just one aspect of the theory of electromagnetism, which also includes classical mechanics and Maxwell's equations. While special relativity can explain how magnetic fields are affected by relative motion, it does not explain the origin of magnetic fields or their behavior in more complex systems.

Similar threads

  • Introductory Physics Homework Help
Replies
7
Views
959
  • Introductory Physics Homework Help
Replies
6
Views
1K
  • Introductory Physics Homework Help
Replies
5
Views
291
  • Introductory Physics Homework Help
Replies
4
Views
1K
  • Introductory Physics Homework Help
Replies
14
Views
1K
  • Introductory Physics Homework Help
Replies
9
Views
1K
  • Introductory Physics Homework Help
Replies
17
Views
374
  • Introductory Physics Homework Help
Replies
9
Views
1K
  • Introductory Physics Homework Help
Replies
2
Views
1K
  • Introductory Physics Homework Help
Replies
4
Views
320
Back
Top